Loooong Proof Game

Loooong Proof Game

Posers and Puzzles

Cookies help us deliver our Services. By using our Services or clicking I agree, you agree to our use of cookies. Learn More.

Secret RHP coder

on the payroll

Joined
26 Nov 04
Moves
155080
15 Aug 06


Proof Game in 40.0 moves
(Position after Black's 40th move. How did the game go?)

B
Non-Subscriber

RHP IQ

Joined
17 Mar 05
Moves
1345
16 Aug 06

Hang on, I'll be right back...

T

Joined
21 Jul 06
Moves
0
16 Aug 06
4 edits

Seems like there's not much market here for these difficult arcane puzzles.

Anyway, to kick off:

1) The last move must have been QxB mate.

2) White has lost six pieces, no pawns.

2) As Black has lost only one pawn and nothing else, this must have been captured on g3 by the a2 pawn. (Not sure about this. Maybe there has been a promotion.)

3) Next question is 'which pawn was it?' But I will leave that for Bowmann.

Immigration Central

tinyurl.com/muzppr8z

Joined
23 Aug 04
Moves
26684
17 Aug 06

Originally posted by ThudanBlunder
Seems like there's not much market here for these difficult arcane puzzles.

Anyway, to kick off:

1) The last move must have been QxB mate.

2) White has lost six pieces, no pawns.

2) As Black has lost only one pawn and nothing else, this must have been captured on g3 by the a2 pawn. (Not sure about this. Maybe there has been a promotion.)

3) Next question is 'which pawn was it?' But I will leave that for Bowmann.
White is technically not in mate yet.

T

Joined
21 Jul 06
Moves
0
17 Aug 06
3 edits

Originally posted by AThousandYoung
White is technically not in mate yet.
Yeah, good job he asked for the past moves, not the next one 😳

M

Joined
12 Mar 03
Moves
44411
17 Aug 06
1 edit

Originally posted by ThudanBlunder
Seems like there's not much market here for these difficult arcane puzzles.

Anyway, to kick off:

1) The last move must have been QxB mate.

2) White has lost six pieces, no pawns.

2) As Black has lost only one pawn and nothing else, this must have been captured on g3 by the a2 pawn. (Not sure about this. Maybe there has been a promotion.)

3) Next question is 'which pawn was it?' But I will leave that for Bowmann.
1) probably. Have you proved yet that it was not a knight taking the bishop on f1, and later the king went to h2 to allow the queen to go to h1 and then the king got back to e1, the queen to f1? Wich would make QxQ+ as last move.

2) white played h2xg3. Probably g2xRh3. And the black a-pawn captured twice (axb and bax to pass the white a-pawn and got promoted. To rook if its the one standing there (most likely). To rook or queen or knight if it got taken later on on g3.

Count the moves black needs at least and see if you can get that below 41. White's moves must accomodate for that.

Secret RHP coder

on the payroll

Joined
26 Nov 04
Moves
155080
18 Aug 06

Originally posted by Mephisto2
Count the moves black needs at least and see if you can get that below 41. White's moves must accomodate for that.
2 - Kb7 (0-0-0 saves a tempo)
4 - Qf1 (three-move routes are blocked)
4 - Ra1 (from d8; Rh8 to a1 is one move slower)
3 - Rb4 (from h8, by elimination)
3 - Ba5 (Pe7 stops the 2-move route)
3 - Ba2 (Pd7 stops the 2-move route)
2 - Nd8
9 - pawns (showing in diagram)
5 - to promote the a-pawn; wPg3 shows a Black promotion occurred,
because the pawn could not get there on its own)
---
35 total moves (so far).

There remain 5 moves to a) play ...NxQd1 or NxBf1 and b) sacrifice a piece on g3.

M

Joined
12 Mar 03
Moves
44411
18 Aug 06

Originally posted by BigDoggProblem
2 - Kb7 (0-0-0 saves a tempo)
4 - Qf1 (three-move routes are blocked)
4 - Ra1 (from d8; Rh8 to a1 is one move slower)
3 - Rb4 (from h8, by elimination)
3 - Ba5 (Pe7 stops the 2-move route)
3 - Ba2 (Pd7 stops the 2-move route)
2 - Nd8
9 - pawns (showing in diagram)
5 - to promote the a-pawn; wPg3 shows a Black promotion occurred,
because ...[text shortened]... s (so far).

There remain 5 moves to a) play ...NxQd1 or NxBf1 and b) sacrifice a piece on g3.
Reasoning further, the promoted knight cannot capture a piece on d1 or f1 AND arrive at g3 to be captured in five moves. So, the pawn could have promoted on b1, from where the route to g3 is possible in 5 moves either via d1 or f1. Check on captures: white misses 6 pieces:
- 3 to get the black a-pawn from a to b to a to b
- 1 to get the black c-pawn on the d-file
- 1 on d1 or f1
- the final captured piece on f1

next question: are the moves feasable, and how? If not, back to the drawing board.

I really hope others jump in here.

Secret RHP coder

on the payroll

Joined
26 Nov 04
Moves
155080
21 Aug 06

Originally posted by Mephisto2
Reasoning further, the promoted knight cannot capture a piece on d1 or f1 AND arrive at g3 to be captured in five moves. So, the pawn could have promoted on b1, from where the route to g3 is possible in 5 moves either via d1 or f1. Check on captures: white misses 6 pieces:
- 3 to get the black a-pawn from a to b to a to b
- 1 to get the black c-pawn on ...[text shortened]... oves feasable, and how? If not, back to the drawing board.

I really hope others jump in here.
Given that Pe7, Pd7 and Ng8 never move, and bQ moves before 0-0-0, there are two routes possible for the black Queen.

1) What are they?
2) Does either collide with the routes of other Black pieces?

M

Joined
12 Mar 03
Moves
44411
21 Aug 06

Originally posted by BigDoggProblem
Given that Pe7, Pd7 and Ng8 never move, and bQ moves before 0-0-0, there are two routes possible for the black Queen.

1) What are they?
2) Does either collide with the routes of other Black pieces?
1) Qd8->a5->h5->h1->f1 and Qd8->b6->h6->h1->f1

2)Both can be played only after the c-pawn has moved but the first two moves of the first one is possible only BEFORE the final b5, g5 and Ba5 (in fact the three moves of that bishop) moves, which makes it very hard. This is not a full proof, but I would concentrate on the second path first.

Secret RHP coder

on the payroll

Joined
26 Nov 04
Moves
155080
21 Aug 06

Originally posted by Mephisto2
1) Qd8->a5->h5->h1->f1 and Qd8->b6->h6->h1->f1

2)Both can be played only after the c-pawn has moved but the first two moves of the first one is possible only BEFORE the final b5, g5 and Ba5 (in fact the three moves of that bishop) moves, which makes it very hard. This is not a full proof, but I would concentrate on the second path first.
Interestingly enough, the Qd8-b6-h6-h1-f1 route doesn't work.

Say Black starts with cxd6 and Qb6. To make further progress, he needs d6-d5 to open the Queen's line to h6. Bb7-d5-a2 must be played before d6-d5, which means b7-b5 must be played before Qd8-b6. However, b7-b5 can't be played until Rh8-h6(h5)-b6(b5)-b4 is played. Unfortunately, bRh8 can't get out, because h7-h5 can't be played until the bQ reaches h1.

Therefore, the Q must travel along the 5th rank, and do it before most of the other Black pieces are developed.

M

Joined
12 Mar 03
Moves
44411
21 Aug 06

Originally posted by BigDoggProblem
Interestingly enough, the Qd8-b6-h6-h1-f1 route doesn't work.

Say Black starts with cxd6 and Qb6. To make further progress, he needs d6-d5 to open the Queen's line to h6. Bb7-d5-a2 must be played before d6-d5, which means b7-b5 must be played before Qd8-b6. However, b7-b5 can't be played until Rh8-h6(h5)-b6(b5)-b4 is played. Unfortunately, bRh8 can ...[text shortened]... t travel along the 5th rank, and do it before most of the other Black pieces are developed.
Yes, figured it out since my last posting.

Nobody seems to join in, despite the fact that you are 'guiding' this exercise. What a shame.

T

Joined
21 Jul 06
Moves
0
21 Aug 06
2 edits

Originally posted by Mephisto2
Nobody seems to join in, despite the fact that you are 'guiding' this exercise. What a shame.
Sorry, long proof games take a lot of time and effort for me, which I can't spare at the moment as I am moving house very soon.
But I posted a couple of nice easier SelfMates.

Secret RHP coder

on the payroll

Joined
26 Nov 04
Moves
155080
22 Aug 06

Originally posted by ThudanBlunder
Sorry, long proof games take a lot of time and effort for me, which I can't spare at the moment as I am moving house very soon.
But I posted a couple of nice easier SelfMates.
I've been trying the s#17, but s# are harder for me to solve than most any other type of chess problem.

Secret RHP coder

on the payroll

Joined
26 Nov 04
Moves
155080
22 Aug 06
1 edit

As Mephisto2 has pointed out, Black will need to promote a Knight on b1.

White is missing QBRRNN. Of the Q and B, one will be taken at home, and the other will be taken at f1 with check on Black's 40th.

This leaves RRNN to sacrifice. Three go to the promoting a7 pawn, and the last piece goes to d6. This last piece has to be Rh1, because Pa7 must promote and go to g3 before the R can get out.

Therefore, Black's first order of business is the promotion of the a-pawn, followed by Qh1. Every other Black move is waiting on these two things to happen, except for Pf7-f3, which can be used as tempo moves if Black gets stuck.

Knowing all this, it's possible to find the first few moves.